Added topic tags to some Giancoli-6 problems + minor typos
[course.git] / latex / problems / Giancoli_6 / problem19.07.tex
1 \begin{problem*}{19.7} % resistor networks
2 A $650\U{\Ohm}$ and a $2200\U{\Ohm}$ resistor are connected in series
3 with a $12\U{V}$ battery.  What is the voltage across the
4 $2200\U{\Ohm0}$ resistor?
5 \end{problem*}
6
7 \begin{solution}
8 First we find the total current in the circuit.  The two resistances,
9 $R_1 = 650\U{\Ohm}$ and $R_2 = 2200\U{\Ohm}$, in series provide an
10 effective resistance of $R_e = R_1 + R_2$.  By Kirchoff's loop rule
11 \begin{align*}
12   V - I R_e &= 0 \\
13   I &= \frac{V}{R_e} = \frac{V}{R_1 + R_2}
14 \end{align*}
15 And applying Ohm's law to the second resistor
16 \begin{align*}
17   V_2 = I R_2 = \frac{V R_2}{R_1 + R_2} = \ans{9.3\U{V}}
18 \end{align*}
19 \end{solution}